Import Question JSON
Current Question (ID: 7505)
Question:
Emission transitions in the Paschen series end at orbit $\text{n} = 3$ and start from orbit $\text{n}$ and can be represented as $\nu = 3.29 \times 10^{15} \left( \frac{1}{3^2} - \frac{1}{\text{n}^2} \right) \text{ Hz}$. The value of $\text{n}$ if the transition is observed at $1285 \text{ nm}$ is:
Solution:
Hint: Use the given formula of frequency
Explanation:
Step 1:
Calculate the frequency from wavelength and (c) velocity of light
Given, $\lambda = 1285 \text{ nm} = 1285 \times 10^{-9} \text{ m}$
We know that the relationship between frequency ($\nu$), wavelength ($\lambda$), and the speed of light ($\text{c}$) is:
$\nu = \frac{\text{c}}{\lambda}$
Using $\text{c} = 3 \times 10^8 \text{ m s}^{-1}$:
$\nu = \frac{3 \times 10^8 \text{ m s}^{-1}}{1285 \times 10^{-9} \text{ m}}$
$\nu \approx 2.33 \times 10^{14} \text{ s}^{-1}$ or $\text{Hz}$
Step 2:
Substitute the value of frequency in the given equation and calculate $\text{n}$.
Given, $\nu = 3.29 \times 10^{15} \left( \frac{1}{3^2} - \frac{1}{\text{n}^2} \right)$
Thus, $2.33 \times 10^{14} = 3.29 \times 10^{15} \left( \frac{1}{3^2} - \frac{1}{\text{n}^2} \right)$
Divide both sides by $3.29 \times 10^{15}$:
$\frac{2.33 \times 10^{14}}{3.29 \times 10^{15}} = \frac{1}{3^2} - \frac{1}{\text{n}^2}$
$0.0708 \approx \frac{1}{9} - \frac{1}{\text{n}^2}$
$0.0708 \approx 0.1111 - \frac{1}{\text{n}^2}$
Rearrange to solve for $\frac{1}{\text{n}^2}$:
$\frac{1}{\text{n}^2} \approx 0.1111 - 0.0708$
$\frac{1}{\text{n}^2} \approx 0.0403$
Take the reciprocal of both sides:
$\text{n}^2 \approx \frac{1}{0.0403}$
$\text{n}^2 \approx 24.81$
Take the square root:
$\text{n} \approx \sqrt{24.81}$
$\text{n} \approx 4.98 \approx 5$
Hence, for the transition to be observed at $1285 \text{ nm}$, $\text{n} = 5$.
The spectrum lies in the infra-red region.
The final answer is $\boxed{2}$
Import JSON File
Upload a JSON file containing LaTeX/MathJax formatted question, options, and solution.